Added topic tags to some Giancoli-6 problems + minor typos
[course.git] / latex / problems / Giancoli_6 / question19.04.tex
1 \begin{problem*}{Q19.4} % resistor networks
2 Two lightbulbs of resistance $R_1$ and $R_2$ ($R_2 > R_1$) are
3 connected in series.  Which is brighter?  What if they are connected
4 in parallel?  Explain.
5 \end{problem*}
6
7 \begin{solution}
8 \Part{a}
9 In series, the same current $I$ flows through both bulbs,
10 so the power (proporional to the brightness) can be found via
11 \begin{align*}
12   R_1 &< R_2 \\
13   I^2 R_1 &< I^2 R_2 \\
14   P_1 = I^2 R_1 &< I^2 R_2 = P_2 \\
15   P_1 &< P_2
16 \end{align*}
17
18 \Part{a}
19 In series, both bulbs see the same voltage $V$, 
20 so the power (proporional to the brightness) can be found via
21 \begin{align*}
22   R_1 &< R_2 \\
23   \frac{1}{R_1} &> \frac{1}{R_2} \\
24   \frac{V^2}{R_1} &> \frac{V^2}{R_2} \\
25   P_1 = \frac{V^2}{R_1} &> \frac{V^2}{R_2} = P_2 \\
26   P_1 &> P_2
27 \end{align*}
28 \end{solution}